LSAT and Law School Admissions Forum

Get expert LSAT preparation and law school admissions advice from PowerScore Test Preparation.

User avatar
 Dave Killoran
PowerScore Staff
  • PowerScore Staff
  • Posts: 5853
  • Joined: Mar 25, 2011
|
#49161
Complete Question Explanation
(The complete setup for this game can be found here: lsat/viewtopic.php?t=9142)

The correct answer choice is (E)

Answer choice (A) is incorrect because F and W cannot be prescribed together.

Answer choice (B) is incorrect because F and V cannot be prescribed together.

Answer choice (C) is incorrect because F and W cannot be prescribed together.

Answer choice (D) is incorrect because if V is selected then M must be selected.

Answer choice (E) is the correct answer choice.
 deck1134
  • Posts: 160
  • Joined: Jun 11, 2018
|
#48885
How do we eliminate answer choice D?

The answer choice to the supplements say that we need M, but I don't know how that is grounds to eliminate this answer.
 deck1134
  • Posts: 160
  • Joined: Jun 11, 2018
|
#48954
Hi PowerScore Staff,

Just bumping this up. I redid the game and this is the only question I can't seem to figure out.

Thanks!
 Administrator
PowerScore Staff
  • PowerScore Staff
  • Posts: 8916
  • Joined: Feb 02, 2011
|
#49162
Hi Deck!

We have posted our official explanation at the top of the thread, and you can view the setup here: lsat/viewtopic.php?t=9142

Get the most out of your LSAT Prep Plus subscription.

Analyze and track your performance with our Testing and Analytics Package.